Linear Transformation involving polynomial and matrix












2














Determine if its linear the transformation



$f:Re_{3} [x]rightarrow M_{2x2}$



such as



$f(ax^3+bx^2+cx+d) = begin{bmatrix}
a-c & 0 \
0&b+d
end{bmatrix}$



for any $ax^3+bx^2+cx+d in Re _{3} [x]$



Im having trouble associating the polynomial with the resulting matrix.
I know I'm supposed to check the two main Linear transformation conditions but I don't know how to aproach this example.



Here's my attempt:



Condition 1: $T(u+v)=T(u)+T(v)$



$f((ax^3+bx^2+cx+d)+(a'x^3+b'x^2+c'x+d'))=f((ax^3+a'x^3)+(bx^2+b'x^2)+(cx+c'x)+(d+d'))=f((a+a')x^3+(b+b')x^2+(c+c')x+(d+d'))=begin{bmatrix}
(a+a')-(c+c') & 0\
0 & (b+b')+(d+d')
end{bmatrix} = begin{bmatrix}
a-c & 0\
0 & b+d
end{bmatrix} + begin{bmatrix}
a'-c' & 0\
0 & b'+d'
end{bmatrix}=f(ax^3+bx^2+cx+d)+f(a'x^3+b'x^2+c'x+d')$



Is this correct?










share|cite|improve this question


















  • 3




    Looks right to me.
    – saulspatz
    2 days ago






  • 3




    Seems fine. ${}{}{}{} $
    – Thomas Shelby
    2 days ago
















2














Determine if its linear the transformation



$f:Re_{3} [x]rightarrow M_{2x2}$



such as



$f(ax^3+bx^2+cx+d) = begin{bmatrix}
a-c & 0 \
0&b+d
end{bmatrix}$



for any $ax^3+bx^2+cx+d in Re _{3} [x]$



Im having trouble associating the polynomial with the resulting matrix.
I know I'm supposed to check the two main Linear transformation conditions but I don't know how to aproach this example.



Here's my attempt:



Condition 1: $T(u+v)=T(u)+T(v)$



$f((ax^3+bx^2+cx+d)+(a'x^3+b'x^2+c'x+d'))=f((ax^3+a'x^3)+(bx^2+b'x^2)+(cx+c'x)+(d+d'))=f((a+a')x^3+(b+b')x^2+(c+c')x+(d+d'))=begin{bmatrix}
(a+a')-(c+c') & 0\
0 & (b+b')+(d+d')
end{bmatrix} = begin{bmatrix}
a-c & 0\
0 & b+d
end{bmatrix} + begin{bmatrix}
a'-c' & 0\
0 & b'+d'
end{bmatrix}=f(ax^3+bx^2+cx+d)+f(a'x^3+b'x^2+c'x+d')$



Is this correct?










share|cite|improve this question


















  • 3




    Looks right to me.
    – saulspatz
    2 days ago






  • 3




    Seems fine. ${}{}{}{} $
    – Thomas Shelby
    2 days ago














2












2








2







Determine if its linear the transformation



$f:Re_{3} [x]rightarrow M_{2x2}$



such as



$f(ax^3+bx^2+cx+d) = begin{bmatrix}
a-c & 0 \
0&b+d
end{bmatrix}$



for any $ax^3+bx^2+cx+d in Re _{3} [x]$



Im having trouble associating the polynomial with the resulting matrix.
I know I'm supposed to check the two main Linear transformation conditions but I don't know how to aproach this example.



Here's my attempt:



Condition 1: $T(u+v)=T(u)+T(v)$



$f((ax^3+bx^2+cx+d)+(a'x^3+b'x^2+c'x+d'))=f((ax^3+a'x^3)+(bx^2+b'x^2)+(cx+c'x)+(d+d'))=f((a+a')x^3+(b+b')x^2+(c+c')x+(d+d'))=begin{bmatrix}
(a+a')-(c+c') & 0\
0 & (b+b')+(d+d')
end{bmatrix} = begin{bmatrix}
a-c & 0\
0 & b+d
end{bmatrix} + begin{bmatrix}
a'-c' & 0\
0 & b'+d'
end{bmatrix}=f(ax^3+bx^2+cx+d)+f(a'x^3+b'x^2+c'x+d')$



Is this correct?










share|cite|improve this question













Determine if its linear the transformation



$f:Re_{3} [x]rightarrow M_{2x2}$



such as



$f(ax^3+bx^2+cx+d) = begin{bmatrix}
a-c & 0 \
0&b+d
end{bmatrix}$



for any $ax^3+bx^2+cx+d in Re _{3} [x]$



Im having trouble associating the polynomial with the resulting matrix.
I know I'm supposed to check the two main Linear transformation conditions but I don't know how to aproach this example.



Here's my attempt:



Condition 1: $T(u+v)=T(u)+T(v)$



$f((ax^3+bx^2+cx+d)+(a'x^3+b'x^2+c'x+d'))=f((ax^3+a'x^3)+(bx^2+b'x^2)+(cx+c'x)+(d+d'))=f((a+a')x^3+(b+b')x^2+(c+c')x+(d+d'))=begin{bmatrix}
(a+a')-(c+c') & 0\
0 & (b+b')+(d+d')
end{bmatrix} = begin{bmatrix}
a-c & 0\
0 & b+d
end{bmatrix} + begin{bmatrix}
a'-c' & 0\
0 & b'+d'
end{bmatrix}=f(ax^3+bx^2+cx+d)+f(a'x^3+b'x^2+c'x+d')$



Is this correct?







linear-transformations






share|cite|improve this question













share|cite|improve this question











share|cite|improve this question




share|cite|improve this question










asked 2 days ago









JakcjonesJakcjones

588




588








  • 3




    Looks right to me.
    – saulspatz
    2 days ago






  • 3




    Seems fine. ${}{}{}{} $
    – Thomas Shelby
    2 days ago














  • 3




    Looks right to me.
    – saulspatz
    2 days ago






  • 3




    Seems fine. ${}{}{}{} $
    – Thomas Shelby
    2 days ago








3




3




Looks right to me.
– saulspatz
2 days ago




Looks right to me.
– saulspatz
2 days ago




3




3




Seems fine. ${}{}{}{} $
– Thomas Shelby
2 days ago




Seems fine. ${}{}{}{} $
– Thomas Shelby
2 days ago










1 Answer
1






active

oldest

votes


















2














That looks fine. Another way to go is this:



Consider the standard basis $$bigl{A_{1,1},A_{2,1},A_{1,2},A_{2,2}bigr}$$ for $M_{2times 2},$ where $A_{i,j}$ is the $2times 2$ matrix of zeroes, except the $i$th row $j$th column entry, which is $1.$ Considering also the standard basis ${x^3,x^2,x,1}$ for $mathfrak{R}_3[x],$ then the given transformation has the matrix representation $$begin{bmatrix}1 & 0 & -1 & 0\0 & 0 & 0 & 0\0 & 0 & 0 & 0\0 & 1 & 0 & 1end{bmatrix}.$$



Since it has a matrix representation, then it's a linear transformation.






share|cite|improve this answer





















    Your Answer





    StackExchange.ifUsing("editor", function () {
    return StackExchange.using("mathjaxEditing", function () {
    StackExchange.MarkdownEditor.creationCallbacks.add(function (editor, postfix) {
    StackExchange.mathjaxEditing.prepareWmdForMathJax(editor, postfix, [["$", "$"], ["\\(","\\)"]]);
    });
    });
    }, "mathjax-editing");

    StackExchange.ready(function() {
    var channelOptions = {
    tags: "".split(" "),
    id: "69"
    };
    initTagRenderer("".split(" "), "".split(" "), channelOptions);

    StackExchange.using("externalEditor", function() {
    // Have to fire editor after snippets, if snippets enabled
    if (StackExchange.settings.snippets.snippetsEnabled) {
    StackExchange.using("snippets", function() {
    createEditor();
    });
    }
    else {
    createEditor();
    }
    });

    function createEditor() {
    StackExchange.prepareEditor({
    heartbeatType: 'answer',
    autoActivateHeartbeat: false,
    convertImagesToLinks: true,
    noModals: true,
    showLowRepImageUploadWarning: true,
    reputationToPostImages: 10,
    bindNavPrevention: true,
    postfix: "",
    imageUploader: {
    brandingHtml: "Powered by u003ca class="icon-imgur-white" href="https://imgur.com/"u003eu003c/au003e",
    contentPolicyHtml: "User contributions licensed under u003ca href="https://creativecommons.org/licenses/by-sa/3.0/"u003ecc by-sa 3.0 with attribution requiredu003c/au003e u003ca href="https://stackoverflow.com/legal/content-policy"u003e(content policy)u003c/au003e",
    allowUrls: true
    },
    noCode: true, onDemand: true,
    discardSelector: ".discard-answer"
    ,immediatelyShowMarkdownHelp:true
    });


    }
    });














    draft saved

    draft discarded


















    StackExchange.ready(
    function () {
    StackExchange.openid.initPostLogin('.new-post-login', 'https%3a%2f%2fmath.stackexchange.com%2fquestions%2f3063041%2flinear-transformation-involving-polynomial-and-matrix%23new-answer', 'question_page');
    }
    );

    Post as a guest















    Required, but never shown

























    1 Answer
    1






    active

    oldest

    votes








    1 Answer
    1






    active

    oldest

    votes









    active

    oldest

    votes






    active

    oldest

    votes









    2














    That looks fine. Another way to go is this:



    Consider the standard basis $$bigl{A_{1,1},A_{2,1},A_{1,2},A_{2,2}bigr}$$ for $M_{2times 2},$ where $A_{i,j}$ is the $2times 2$ matrix of zeroes, except the $i$th row $j$th column entry, which is $1.$ Considering also the standard basis ${x^3,x^2,x,1}$ for $mathfrak{R}_3[x],$ then the given transformation has the matrix representation $$begin{bmatrix}1 & 0 & -1 & 0\0 & 0 & 0 & 0\0 & 0 & 0 & 0\0 & 1 & 0 & 1end{bmatrix}.$$



    Since it has a matrix representation, then it's a linear transformation.






    share|cite|improve this answer


























      2














      That looks fine. Another way to go is this:



      Consider the standard basis $$bigl{A_{1,1},A_{2,1},A_{1,2},A_{2,2}bigr}$$ for $M_{2times 2},$ where $A_{i,j}$ is the $2times 2$ matrix of zeroes, except the $i$th row $j$th column entry, which is $1.$ Considering also the standard basis ${x^3,x^2,x,1}$ for $mathfrak{R}_3[x],$ then the given transformation has the matrix representation $$begin{bmatrix}1 & 0 & -1 & 0\0 & 0 & 0 & 0\0 & 0 & 0 & 0\0 & 1 & 0 & 1end{bmatrix}.$$



      Since it has a matrix representation, then it's a linear transformation.






      share|cite|improve this answer
























        2












        2








        2






        That looks fine. Another way to go is this:



        Consider the standard basis $$bigl{A_{1,1},A_{2,1},A_{1,2},A_{2,2}bigr}$$ for $M_{2times 2},$ where $A_{i,j}$ is the $2times 2$ matrix of zeroes, except the $i$th row $j$th column entry, which is $1.$ Considering also the standard basis ${x^3,x^2,x,1}$ for $mathfrak{R}_3[x],$ then the given transformation has the matrix representation $$begin{bmatrix}1 & 0 & -1 & 0\0 & 0 & 0 & 0\0 & 0 & 0 & 0\0 & 1 & 0 & 1end{bmatrix}.$$



        Since it has a matrix representation, then it's a linear transformation.






        share|cite|improve this answer












        That looks fine. Another way to go is this:



        Consider the standard basis $$bigl{A_{1,1},A_{2,1},A_{1,2},A_{2,2}bigr}$$ for $M_{2times 2},$ where $A_{i,j}$ is the $2times 2$ matrix of zeroes, except the $i$th row $j$th column entry, which is $1.$ Considering also the standard basis ${x^3,x^2,x,1}$ for $mathfrak{R}_3[x],$ then the given transformation has the matrix representation $$begin{bmatrix}1 & 0 & -1 & 0\0 & 0 & 0 & 0\0 & 0 & 0 & 0\0 & 1 & 0 & 1end{bmatrix}.$$



        Since it has a matrix representation, then it's a linear transformation.







        share|cite|improve this answer












        share|cite|improve this answer



        share|cite|improve this answer










        answered 2 days ago









        Cameron BuieCameron Buie

        85.1k771155




        85.1k771155






























            draft saved

            draft discarded




















































            Thanks for contributing an answer to Mathematics Stack Exchange!


            • Please be sure to answer the question. Provide details and share your research!

            But avoid



            • Asking for help, clarification, or responding to other answers.

            • Making statements based on opinion; back them up with references or personal experience.


            Use MathJax to format equations. MathJax reference.


            To learn more, see our tips on writing great answers.





            Some of your past answers have not been well-received, and you're in danger of being blocked from answering.


            Please pay close attention to the following guidance:


            • Please be sure to answer the question. Provide details and share your research!

            But avoid



            • Asking for help, clarification, or responding to other answers.

            • Making statements based on opinion; back them up with references or personal experience.


            To learn more, see our tips on writing great answers.




            draft saved


            draft discarded














            StackExchange.ready(
            function () {
            StackExchange.openid.initPostLogin('.new-post-login', 'https%3a%2f%2fmath.stackexchange.com%2fquestions%2f3063041%2flinear-transformation-involving-polynomial-and-matrix%23new-answer', 'question_page');
            }
            );

            Post as a guest















            Required, but never shown





















































            Required, but never shown














            Required, but never shown












            Required, but never shown







            Required, but never shown

































            Required, but never shown














            Required, but never shown












            Required, but never shown







            Required, but never shown







            Popular posts from this blog

            Mario Kart Wii

            What does “Dominus providebit” mean?

            Antonio Litta Visconti Arese